Howdy, Stranger!

It looks like you're new here. If you want to get involved, click one of these buttons!

CBF, MBF distinction in Logic Games

BumblebeeBumblebee Member
in Logic Games 640 karma

Hello 7sagers,

If a question in LG asks, "Which one of the following Could Be False?"

Can a choice that is Must Be False be correct?

Similarly, if a question in LG asks, "Which of the following Could Be True?"

Can a choice that is Must Be True be correct?

Thanks very much in advance!

Comments

  • cstrobelcstrobel Alum Member
    228 karma

    I believe yes to both, because if it must then it could.
    Especially because this is for LG, there will only be one, eventually clear-cut, answer. Because of that, if you know that it must then you can pounce on that as the right answer.

  • akistotleakistotle Member 🍌🍌
    edited March 2018 9372 karma

    @LSATlife said:
    Hello 7sagers,

    If a question in LG asks, "Which one of the following Could Be False?"

    Can a choice that is Must Be False be correct?

    Similarly, if a question in LG asks, "Which of the following Could Be True?"

    Can a choice that is Must Be True be correct?

    Thanks very much in advance!

    Yes, to both questions! This happens a lot.

  • BumblebeeBumblebee Member
    640 karma

    Thank you @cstrobel and @akistotle! :)

  • Cant Get RightCant Get Right Yearly + Live Member Sage 🍌 7Sage Tutor
    27821 karma

    For sure. What Game/question are you referring to? I’ve never paid attention to this; once I see a “could be” AC I don’t stick around to consider it any further. Though they absolutely could do this and the AC would be unambiguously correct, I’m just a little surprised they actually would.

  • BumblebeeBumblebee Member
    640 karma

    @"Cant Get Right" Hi Josh! I was solving PT36 Game 4, which contains two CBF questions. These questions actually don't contain MBF as correct answers (CBF AC's are correct!), but as I was reviewing the meaning of the question stem carefully, I thought of a hypothetical situation where I may be given a MBF answer choice for a CBF question and wondered whether I could choose MBF as the correct AC. Along with that thought, I wondered if we are ever given both MBF and CBF AC's to a CBF question, whether there is a reason to choose the CBF AC over MBF AC.

    Based on the help I received above, it must be the case that a CBF cannot have both CBF and MBF AC's because there can't be two correct answer choices to a question.

    My takeaway is that if a CBF does contain a MBF as one of the AC's, that MBF AC would be correct. Thanks so much! :) :)

Sign In or Register to comment.